LSAT 7 – Section 4 – Question 23

You need a full course to see this video. Enroll now and get started in less than a minute.

Target time: 1:06

This is question data from the 7Sage LSAT Scorer. You can score your LSATs, track your results, and analyze your performance with pretty charts and vital statistics - all with a Free Account ← sign up in less than 10 seconds

Question
QuickView
Type Tags Answer
Choices
Curve Question
Difficulty
Psg/Game/S
Difficulty
Explanation
PT7 S4 Q23
+LR
Sufficient assumption +SA
A
3%
159
B
2%
156
C
63%
166
D
20%
159
E
11%
161
144
157
171
+Harder 150.348 +SubsectionHarder
This page shows a recording of a live class. We're working hard to create our standard, concise explanation videos for the questions in this PrepTest. Thank you for your patience!

This is a sufficient assumption question, as indicated by the question stem: The conclusion above is properly drawn if which one of the following is assumed?

This is a very conditional-heavy stimulus.

Premise 1: /strike → increase wages

Premise 2: increase wages → sell sub

Conclusion: sell sub

Remember, conditionals have to be triggered in order for us to conclude anything from them. Since we’re trying to prove that Bell’s subsidiaries will be sold, we need something to trigger the first or second premise: either that there will not be a strike or that the wages will be increased.

Answer Choice (A) This does have anything to do with our premise or conclusion and it does help validate our conclusion. It doesn’t trigger our conditionals either.

Answer Choice (B) This is negating the necessary condition in the first premise, which would trigger backward, allowing us to conclude that the workers will do on strike. This answer choice also negates the sufficient condition in premise 2, which renders the rule useless. This is incorrect.

Correct Answer Choice (C) This triggers the first premise, which also triggers our second premise, leading to the conclusion that Bell’s subsidiaries will be sold.

Answer Choice (D) This AC is saying the president can increase wages; this does not mean that he will.

Answer Choice (E) This is additional information that does not clearly trigger our conditionals. The workers only wanting a wage increase does not help us validate the conclusion.

Take PrepTest

Review Results

Leave a Reply